Download as pdf or txt
Download as pdf or txt
You are on page 1of 74

EVIDENCE

PROFESSOR VINCENT ALEXANDER

Copyright © 2016 by BARBRI, Inc.


EVIDENCE—MULTISTATE

Prof. Vincent C. Alexander


St. John’s University
School of Law
New York

INTRODUCTION

A. SOURCES OF EVIDENCE LAW


Federal Rule of Evidence
Multistate Law: ___________________________________________________

B. OVERVIEW: EVIDENCE TOPICS

1. The Three Major Topics


Relevance
(a) ______________________________
Witness
(b) ______________________________
Hearsay
(c) ______________________________

2. The Three Minor Topics


Authentication
(a) ______________________________
Best evidence rule
(b) ______________________________
Privileges
(c) ______________________________

3. Five Topics Not Covered in Lecture

(a) Procedural Considerations


(b) Burden of Proof
(c) Presumptions
(d) Judicial Notice
(e) Real Evidence

Copyright © 2016 by BARBRI, Inc.


2. EVIDENCE

I. RELEVANCE

A. Basic Principles
any tendency
1. Evidence is RELEVANT if it has _______________________________ to
make a material fact more probable or less probable than would be the case
without the evidence.

2. All relevant evidence is ADMISSIBLE, UNLESS (a) some specific


privilege,hearsay EXCLUSIONARY RULE is applicable, or (b) the court makes a discretionary
determination that the probative value of the evidence is substantially
outweighed by one or more of six PRAGMATIC CONSIDERATIONS:
Danger of unfair prejudice
Confusion of issues
__________________________
Misleading the jury
——accuracy
__________________________
—effeciency

__________________________
undue delay
waste of time
unduly accumulative
__________________________

__________________________

__________________________

B. Similar Occurrences

IN GENERAL, if evidence concerns SOME TIME, EVENT, OR PERSON


OTHER THAN THAT INVOLVED IN THE CASE AT HAND, the evidence is
INADMISSIBLE. Probative value is usually outweighed by pragmatic
considerations (e.g., weak relevance, danger of confusion, misleading the jury,
time-consuming). But some recurring situations have produced concrete rules
that may permit admissibility.
EVIDENCE 3.

1. Plaintiff’s Accident History.


inadmissible
GENERALLY, plaintiff’s accident history is _________________ because it
shows nothing more than the fact that the plaintiff is accident-prone.
Being accident-prone is nothing more than character evidence, not
allowed in civil action to prove a person's conduct on a particular
______________________________________________________
occasion.

______________________________________________________

EXCEPTION: Plaintiff’s prior accidents ADMISSIBLE if:


the event that caused the plaintiff injury is in issue
_______________________________________________________

HYPO 1. Billy Joel drove into a lamp-post and sues the municipality in negligence, alleging
that the placement of the post created a hazardous condition. (a) On the issue of contributory
negligence, should the municipality be allowed to introduce evidence that Billy has frequently
driven into other stationary objects (tree, bridge, brick wall)?
no, Billy’s propensity or character for carelessness
(b) What if Billy claims that the accident injured his shoulder, and the municipality wants to
show that Billy’s shoulder was injured when he crashed into a tree a year before the lamp-post
incident?
exception: caused by another accident

Q-TIP: Always ask yourself—For what purpose is the evidence being offered?

2. Similar Accidents Caused by Same Instrumentality or Condition.

GENERALLY, other accidents involving defendant are inadmissible because they


suggest nothing more than general character for carelessness.

EXCEPTION: Other accidents involving the same instrumentality or condition


may be admitted for three potential purposes IF the other accident occurred
under substantially similar circumstances
_______________________________________:
To show the existence of
dangerous condition
(1) ____________________
any one of them will be sufficient
To show causation of the
accident
(2) ___________________

Prior notice to the


defendant
(3) ___________________
4. EVIDENCE

HYPO 2. Assume in Hypo 1 that several other vehicles had collided with the same lamp-post
that Billy ran into. Could Billy introduce those other accidents against the municipality?
Yes, if those other accidents happened under
substantial similar circumstances, like weather,
lighting, traffic conditions.

Substantial similarity rule also governs the admissibility of EXPERIMENTS and


TESTS, i.e., tests or experiments must be conducted under circumstances
substantially similar to the matter at issue.

3. Intent in Issue.

Prior similar conduct of a person may be admissible to raise an inference of the


person’s intent on a later occasion.

HYPO 3. Paris sues Brewski Co. for gender discrimination, alleging that she was qualified for
the job but was not hired because of her gender. She seeks to show that Brewski hired no
women, despite their qualifications, during the past six years. Admissible?
Yes, Breeski’s treatment of other qualified women tends
to show discriminatory intent with respect to Paris.

4. Comparable Sales on Issue of Value.

Selling price of other property of similar type, in same general location, and close
in time to period at issue, is some evidence of value of property at issue.
EVIDENCE 5.

5. Habit.

Habit of a person (or routine of a business organization) is ADMISSIBLE as


circumstantial evidence of how the person (or business) acted on the occasion at
issue in the litigation.

DISTINGUISH: Character evidence refers to a person’s general disposition or


propensity. Character is usually not admissible to prove conduct on a particular
occasion. E.g., Fact that Carlos is a “careless” driver is inadmissible to suggest
that he ran a red light and caused the accident involving the plaintiff.

DEFINITION: Habit is a repetitive response to a particular set of circumstances.


Habit has two defining characteristics:
1. Frequency of the conduct
2 Particularity of the conduct
_______________________

_______________________

HYPO 4. In an auto accident case, the issue is whether Lindsay stopped her car at the stop sign
at the intersection of Hickory and Main Streets.

(a) Plaintiff calls Britney to testify that during the six months preceding the accident, she
had seen Lindsay run red lights, change lanes without using signals and run stop signs
throughout town. Admissible as habit evidence to prove that Lindsay ran the stop
sign at Hickory and Main?
No, prior acts of Lindsay tend to show mere general
carelessness

failed particularity test

(b) Britney will testify that she has seen Lindsay run the stop sign at Hickory and Main
on at least eight occasions within a two-week period. Admissible as habit?
Yes, frequently repeated response, time, particular
set of circumstances
6. EVIDENCE

KEY WORDS: “always,” “never,” “invariably,” automatically,” “instinctively.”

Business Routine: Example: To prove that a particular letter was mailed by


CEO, evidence that CEO put letter in her out-box on Tuesday, and messenger
“routinely” picks up mail in CEO’s out-box at 3:00 P.M. each business day for
delivery to mail room.

6. Industrial Custom as Standard of Care.

Evidence as to how others in the same trade or industry have acted in the recent
past may be admitted as some evidence as to how a party in the instant litigation
should have acted, i.e., as evidence of the APPROPRIATE STANDARD OF
CARE.

Example: Plaintiff is injured when a blade spins off a lawn-mower. In an action


against the manufacturer, she may show that 80% of all other lawn-mower
manufacturers, during the relevant time period, had installed devices to prevent
blade spin-off.
standard of care in lawn-mower industry

C. Policy-Based Exclusions

1. Liability Insurance. p.57

Evidence that a person has (or does not have) liability insurance is
inadmissible
_______________ proving fault or the absence of fault
for the purpose of _______________________________
Policy: to avoid risk that jury will base decision on availability of insurance
instead of merits of case, and to encourage purchase of liability insurance.
admissible
EXCEPTION: Evidence of insurance may be _________________ for some
other relevant purpose, such as: (a) proof of OWNERSHIP / CONTROL OF
that issue is disputed by defendant
INSTRUMENTALITY OR LOCATION, IF _________________

impeachment
or (b) for the purpose of _________________of a witness (the process of trying to
show that a witness should not be believed). on the ground by bias
EVIDENCE 7.

HYPO 5. Rosie fell down a well on Trump’s property. Rosie sues Trump, contending that the
well was impossible to see because of overgrown foliage. Trump denies that he was negligent
and also defends, in the alternative, on the ground that he did not own the land in question.

(a) Should Rosie be allowed to introduce evidence that Trump carried a homeowner’s
liability insurance policy on the land?
as evidence of Trump’s negligence, not admissible
To show Trump’s ownership of the land, Yes

(b) Same case. Apprentice, a witness called by Trump, testifies that she had been on
Trump’s property just prior to the accident and there was no foliage covering the
well. May Rosie show, during cross-examination of Apprentice, that Apprentice is a
claims adjuster employed by the company that issued the homeowner’s policy to
Trump?
Yes. To impeach a witness on the ground of bias.

LIMITING INSTRUCTION should be given to the jury whenever evidence is admissible


for one purpose but not for another. Judge should tell jury to consider the evidence only
for the permissible purpose.

2. Subsequent Remedial Measures. p.57 party admission

Post-accident repairs, design changes, policy changes.


Subsequent inadmissible
remedial measures
________________ for the purpose of proving NEGLIGENCE,
CULPABLE CONDUCT, PRODUCT DEFECT, or NEED FOR
WARNING. Policy: To encourage post-accident repairs, etc. to avoid
future accidents.
8. EVIDENCE

admissible
EXCEPTION: Subsequent remedial measures may be _____________
for some other relevant purpose, such as proof of OWNERSHIP /
CONTROL or FEASIBILITY OF SAFER CONDITION, IF EITHER IS
disputed by the defendant
________________________.

HYPO 6. Penelope bought a cup of coffee at Dante’s Coffee Inferno and scalded her tongue
because the coffee was too hot. She sues Dante’s in negligence. Dante’s denies that it was
negligent.

(a) At trial, Penelope seeks to introduce evidence that after the accident, Dante’s installed
new thermostats on its coffee-brewing equipment. Penelope contends that this
conduct is an admission by Dante’s that better safety controls were feasible.
Admissible?
No, basic exclusionary rule applies
Dante’s new thermostats are subsequent remedial
measure, no good to prove negligence.
Still no good to show feasible. Because feasibility to
do sth was not disputed by the defendant.

(b) Same case, except now assume that Penelope contends that Dante’s negligence
consisted of the failure to place warnings on its coffee cups indicating that its coffee
was too hot for human consumption. Dante’s defends, in part, on the ground that it
was impossible to affix labels to its coffee cups. Penelope seeks to introduce
evidence that after the accident, Dante’s began to use cups that were pre-printed with
warnings. Admissible?
Yes. Defendant is disputing the feasibility of doing
sth safer.

NOTE: In a products liability action based on strict liability, the


manufacturer’s subsequent remedial measures are inadmissible to show
the existence of a defect in the product at the time of the accident. The
federal (multistate) rule differs from the rule in some states.
EVIDENCE 9.

p.57
3. Settlements of Disputed Civil Claims.
disputed civil claim
In the event of a __________________________,

the following are INADMISSIBLE


settlement
offer to settle
____________________________
statement of fact made during statement discussions

____________________________

____________________________
showing liability, impeaching a witness as prior
inconsistent statement
for the purpose of ______________________________________.

Policy: To encourage settlement.

EXCEPTIONS:

(1) Settlement evidence admissible for the purpose of


impeaching a witness on the ground of bias
_________________________________

(2) Statements of fact made during settlement discussion in civil litigation


government regulatory agency
with a ____________________________________ are admissible in a
criminal
later ___________________ (e.g., corporate fraud case in which
corporate officers make admissions of fact during civil settlement talks
with the SEC and are later prosecuted for crimes based on the same
facts). Rationale: public policy favors prosecutor’s use of highly
probative factual evidence.

Note: The exception in criminal cases does not apply to settlements


and offers to settle.

HYPO 7. On their way home form the Jersey Shore, Vinny and Pauly were simultaneously
struck by a car being driven by Snooki. Vinny and Pauly both filed suit against Snooki, each
seeking $100,000. Snooki denied all allegations.

(a) Before trial, Vinny settled with Snooki for $50,000. When Pauly’s case went to trial,
Pauly sought to introduce the Vinny-Snooki settlement as evidence that Snooki, in
effect, acknowledged her fault. Admissible?
No. not for liability
10. EVIDENCE

(b) Before Pauly’s case went to trial, Pauly and Snooki met to discuss possible
settlement. During the discussion, Pauly said, “I’ll accept $50,000 in settlement. So
what if I was jay-walking?” Snooki declined. At trial, should Snooki be allowed to
introduce (1) Pauly’s offer to settle and (2) Pauly’s admission that he was jay-
walking?
(1)no. offer to settle cannot be used to show the
weakness of offeror’s case
(2)statement of fact made during a settlement
discussion.

If Pauly testified at the trial that he did not jay-walk, could Snooki introduce his
prior inconsistent statement to impeach his credibility?
No. Statement of fact made during settlement cannot
be used to impeach credibility of inconsistent
statement. Policy value outweighs impeachment.

(c) At the trial of Pauly’s case, Snooki called Vinny as a witness and Vinny testified to
the effect that Snooki did not drive negligently. On cross-examination of Vinny,
should Pauly be allowed to prove the Vinny-Snooki settlement?
Yes, as evidence of Vinny’s bias.

NOTE: The exclusionary rule only applies if there is a CLAIM that is


DISPUTED (at time of settlement discussion) either as to validity of the
claim or the amount of damages. liability

HYPO 8(a). A’s and B’s cars collided. B immediately ran up to A and said, “Look, I’ll settle
with you for $100,000 if you don’t sue.” Should A be allowed to introduce B’s statement against
him at a subsequent trial?
Yes, it’s relevant. Admission of fault by B.
A has not yet asserted any claim against B at the time.
Rule is not triggered.
EVIDENCE 11.

HYPO 8(b). After A’s and B’s cars collided, A sent a letter to B saying, “The accident was all
your fault. I demand that you pay my damages in the amount of $100,000.” B called A on the
phone and said, “You’re right about the accident. It was all my fault and I owe you the full
$100,000 you’re asking for. But you know how fickle juries can be. If you don’t accept $50,000
now, you’ll have to sue me to get anything.” Should A be allowed to introduce B’s statements
against B at a subsequent trial?

Yes, B has not disputed liability or damages at the time of


discussion.

Variation on 8(b): What if B had said, “It was all my fault, but you didn’t suffer $100,000 in
damages”?
Enough to trigger the exclusionary rule.

HYPO 9. Arnold sold toxic “Terminator” action figures to the public. The Environmental
Protection Agency (EPA) sued Arnold for civil penalties. Arnold denied liability. In
negotiations with the EPA, Arnold offered to settle for half the amount sought, admitting during
the discussions that the toys were toxic. Thereafter, the Government prosecuted Arnold for
violating criminal laws against the distribution of toxic toys. In the criminal case, as evidence of
Arnold’s guilt, should the Government be allowed to introduce:

(A) Arnold’s offer to settle with the EPA?


No.

(B) Arnold’s admission to the EPA that the toys were toxic?
Yes. statements of facts made during settlement talk with
gov regulatory agency.
12. EVIDENCE

Plea Bargaining in Criminal Cases: The following are INADMISSIBLE:

Offer to plead guilty—cannot be used against the defendant in the pending


criminal case or in subsequent civil litigation based on the same facts.

Withdrawn guilty plea—cannot be used against the defendant in the


pending criminal case or in subsequent civil litigation based on the same
facts.

Plea of nolo contendere (“no contest”)—cannot be used against the


defendant in subsequent civil litigation based on the same facts.

Statements of fact made during any of the above plea discussions.

BUT, a plea of GUILTY (not withdrawn) is ADMISSIBLE in subsequent


litigation based on the same facts under the rule of party admissions.
p.57

4. Offer to Pay Hospital or Medical Expenses.

Evidence that a party has paid or offered to pay an accident victim’s


inadmissible
hospital or medical expenses is ________________ to prove liability.
Policy: To encourage charity.

NOTE: This rule does not exclude other statements made in connection
with an offer to pay hospital or medical expenses.

HYPO 10. Donna’s car hit pedestrian Pablo. Donna immediately ran to Pablo and said, (a)
“Don’t worry about a thing. I’ll pay for your hospital bills. (b) I’m sorry I ran the red light.”

(a) Is statement (a) admissible against Donna?


No. offer to pay

(b) Is statement (b) admissible against Donna?


Yes, outright admission of fact by defendant. Statements
of made in connection with offer to pay medical expenses
are admissible.
EVIDENCE 13.

D. Character Evidence

Character evidence refers to a person’s general propensity or disposition, e.g., honesty,


fairness, peacefulness, or violence.

Potential purposes for the admissibility of character evidence:

(1) Person’s character is an ESSENTIAL ELEMENT IN THE CASE.


rare in civil, never in criminal cases

(2) Character evidence AS CIRCUMSTANTIAL EVIDENCE OF THE


PERSON’S CONDUCT ON A PARTICULAR OCCASION.

(3) Witness’s bad character for truthfulness to IMPEACH CREDIBILITY.


p.42

1. Criminal Cases.

(a) Defendant’s Character.

Overview: Evidence of the defendant’s character to prove defendant’s


is inadmissible
conduct on a particular occasion ________________________________
DURING THE PROSECUTION’S CASE-IN-CHIEF.

However, DEFENDANT, during the defense, may introduce


evidence of relevant character trait
________________________________________________________
(by reputation or opinion testimony of a character witness) to prove his
conduct, which OPENS THE DOOR TO REBUTTAL by the prosecution.

HYPO 11. Rambo is charged with murder. During its direct case, should the prosecution be
allowed to introduce evidence that Rambo has been convicted three times for assault, has a bad
reputation for violence, and he recently stampeded a herd of cattle through the middle of town?
No. Just show the general propensity of violence. Not allowed.
14. EVIDENCE

Should the prosecution’s proposed evidence be admitted on the ground that defendant’s violent
character is an essential element of the crime with which Rambo is charged?

No. violence character is not an essential element of a


crime.

HYPO 12. During the defense, Rambo calls Trautman to the stand to testify: (1) “I’m familiar
with Rambo’s reputation for peacefulness, and it is excellent. (2) I personally know Rambo, and
in my opinion he is a peaceful person.” Admissible? For what purpose?

Yes. Admission, switch to the defendant side. It is an option.


relevant character trait as circumstantial evidence as good
conduct.

When character evidence is admissible through a character witness to


prove conduct in conformity, the proper form:
reputation, opinion
______________________

______________________

Could Trautman properly testify: “I’ve seen Rambo turn the other cheek
when assaulted by bullies; just last week he was elected President of the
local Pacifist Club.”
No, evidence of specific acts

Could Trautman properly testify, “Rambo’s reputation for bravery and


honesty is excellent”?
No, not relevant character trait here. It should
be peacefulness.

The character witness can only testify to a


character trait that is relevant to the type of
crime which the defendant is charged.

Law-abiding in general is Ok.But,it is deemed to


apply to trait at issue.
EVIDENCE 15.

(b) Prosecution’s Rebuttal.

IF defendant has “opened the door” by calling character witnesses, the


prosecution may rebut:

(1) by cross-examining defendant’s character witnesses with


“Have you heard” or “Did you know” questions about specific
acts of the defendant that reflect adversely on the particular
character trait that defendant has introduced (prosecution must
have good faith basis for the question); limited purpose: to
impeach character witness’ knowledge about the defendant;
and/or

(2) by calling its own reputation or opinion witnesses to contradict


defendant’s witnesses.

HYPO 13. During the defense, Rambo called Trautman to testify to Rambo’s peaceful
character.

(a) Could the prosecutor ask Trautman, on cross-examination, (i) “Have you heard that
Rambo was arrested last year for assaulting Rocky?” (ii) “Did you know that Rambo
shot Judge Dredd three years ago?”
Yes. P is allowed to test the character witness knowledge
about Ramboo reputation and to test soundness.

(b) If Trautman denies having heard or knowing of the arrests or bad acts mentioned by
the prosecutor, may the prosecutor prove that they actually occurred?
No, P must take the answer of the character
witness(bound by the witness answer), not allowed to
prove. Not worthy, time-consuming.

Inquiry.
16. EVIDENCE

(c) Could the prosecutor properly ask Trautman, “Have you heard (or did you know) that
Rambo cheated on his income taxes last year?”
No, irrelevant bad act. Only relevant.

Even law-abiding, keep that narrow, deemed to apply only


to the character trait in issue.

HYPO 14. Assume Rambo introduced Trautman’s favorable character testimony. After the
defense rests, the prosecution calls Murdock to testify that he has known Rambo for 20 years, is
familiar with Rambo’s reputation for peacefulness in the community, and that such reputation is
bad. Rambo’s attorney objects on the ground that this is impermissible character evidence.
Objection overruled.
During the prosecution rebuttal case, the P can call her own
character witness to give reputation and opinion testimony
to rebut, contradict, neutralize the defendants character
witness.

HYPO 15. Assume that the only witness who testified during the defense was Rambo himself,
and he testified only to the fact that he did not commit the murder. After the defense rests, the
prosecution calls Murdock to testify that Rambo has a reputation for violence. Rambo’s attorney
objects on the ground that this is impermissible character evidence.
Objection sustained. In the case, Rambo did not open the
door.

Only way to open door in exam: defendant calls character


witness
But there is some permissible ways to impeach a
defendant’s credibility as a witness. Credibility for
truthfulness if they take the stand. That’s separate
issue.
(c) Victim’s Character—Self-Defense Case.
Criminal only
In addition to direct evidence that the alleged victim of an assault was the first
aggressor (e.g., witness testifies, “I saw victim aim a gun at defendant”), the
criminal defendant may introduce evidence of victim’s violent character as
circumstantial evidence that the victim was the first aggressor.
EVIDENCE 17.

Proper method: character witness may testify to victim’s reputation for violence
and may give opinion.

Prosecution rebuttal: evidence of victim’s good character for peacefulness (with


reputation or opinion). In addition, prosecution may prove defendant’s character
for violence.

Homicide: If defendant offers evidence of any kind that victim was the first
aggressor, prosecution may introduce evidence of victim’s good character for
peacefulness. Example: D’s witness testifies, “Victim aimed a gun at D.”

HYPO 16. Defendant, Coach Bobby, has been charged with assault for throwing a chair at
Tonya. Coach Bobby claims that Tonya started the fight and lunged at him with a knife. To
prove that Tonya was the first aggressor, Bobby calls Nancy to testify:

(a) That she knows Tonya and that in her opinion, Tonya is a very violent woman.
Admissible. Criminal self-defense case. Victim bad
character. Circumstantial evidence that Tonya is
the first aggressor.

(b) That she (Nancy) had been the victim of a knife attack by Tonya a few years ago.
No, evidence of specific act. Only permissible way:
reputation and opinion.

(c) What if Bobby offers evidence that, at the time of the altercation with Tonya, he was
aware of her prior knife attack on Nancy?
Admissible. Even it’s a evidence of specific act.

Separate rule of relevance: If the defendant, at the time of the alleged self-
defense, was aware of the victim’s violent reputation or prior specific acts of
violence, such awareness may be proven to show the defendant’s state of mind—
fear—to help prove that he acted reasonably in responding as he did to the
victim’s aggression.
18. EVIDENCE

(d) Victim’s Character—Sexual Misconduct Case.

Under “rape shield law,” in both criminal and civil cases, where defendant
is alleged to have engaged in sexual misconduct, the following evidence
about the victim is ordinarily inadmissible:

opinion or reputation evidence about the victim’s sexual


propensity,

evidence of specific sexual behavior of the victim

Exceptions in Criminal Cases:

(1) specific sexual behavior of the victim to prove that someone


other than the defendant was the source of semen or injury to
the victim; sb else did it.

(2) victim’s sexual activity with the defendant if the defense of


consent is asserted; or
prior sexual activity between victim and defendant himself
(3) where exclusion would violate defendant’s right of due
process. Example: “Love Triangle Defense.” Defendant
should be allowed to show that the victim had a sexual
relationship with X at the time of defendant’s alleged rape if X
was aware of victim’s sexual contact with defendant. Purpose:
to suggest the victim had a motive to falsely claim that the
sexual contact with defendant was nonconsensual.

Exception in Civil Cases:

The court may admit evidence of specific sexual behavior or sexual


propensity of the victim if its probative value substantially outweighs the
danger of harm to the victim and unfair prejudice to any party.

2. Civil Cases.

(a) Character evidence generally INADMISSIBLE to prove a


person’s conduct on a particular occasion.

HYPO 17. A sues B for automobile negligence.

(a) During the plaintiff’s case-in-chief, A seeks to offer evidence of B’s reputation for
careless driving. Admissible?
No.
EVIDENCE 19.

(b) During the defense, B calls Witness to testify that, in her opinion, B is a prudent and
careful driver. Admissible?
No.

HYPO 18. Nicole’s estate sues OJ for wrongful death damages, alleging that OJ intentionally
killed Nicole. During the defense, may OJ properly introduce evidence of his peaceful
character?
No, civil action.Even when the underlying conduct is
criminal in nature, we don’t allow a party in civil case
to show the character.

(b) Evidence of person’s character is ADMISSIBLE in civil action where


such character is an ESSENTIAL ELEMENT OF A CLAIM OR
DEFENSE (provable by reputation, opinion, and specific acts). Only
three situations:
Tort action alleging neligent hiring or entrustment;
Tort of defamation;
____________________________
Child cutody dispute.

____________________________

____________________________

Example (1): P was struck in 2012 by a truck being driven by Charlie,


who was acting within the scope of his employment for Acme Trucking.
P sues Acme, alleging that Acme was negligent in hiring Charlie in 2011
and thereafter allowing him to drive on Acme’s behalf. (Tort theory:
Acme knew or should have known that Charlie was an accident risk.) P
may introduce the testimony of character witnesses that Charlie had a
reputation for being a careless driver, and they have a low opinion of
Charlie’s care in driving. P may also prove that Charlie had been involved
in three prior accidents.

Example (2): P sues Newspaper for libel based on a story in which P was
accused of being dishonest. To support its defense of “truth,” Newspaper
may introduce reputation, opinion and specific-act evidence about P’s
dishonesty; and P may use the same type of evidence to show P’s honesty.

Example (3): In a matrimonial dispute, H and W contest the custody of


child X. On the fitness of each parent to have custody, evidence is
allowed of each parent’s relevant acts and reputation as well as opinion
evidence bearing on their fitness.
20. EVIDENCE

E. Defendant’s Other Crimes for Non-Character Purpose

General rule: Other crimes or specific bad acts of defendant are not admissible during
the prosecution’s case-in-chief if the only purpose is to suggest that because of
defendant’s bad character he is more likely to have committed the crime currently
charged.

Example: D is charged with robbing bank A. The fact that D robbed bank B six months
later would be inadmissible character evidence.

BUT, defendant’s bad acts or other crimes may be admissible to show:


something specific about the crime currently charged
______________________________________________________________

(something separate and apart from mere propensity to commit the crime.)

The five most common non-character purposes are: “MIMIC”—


Motive
Intent
_________________________
Mistake or accident, the absence thereof
Identity
Common scheme or plan
_________________________

_________________________

_________________________

_________________________

NOTE: If a MIMIC category is satisfied, the prosecution may use other-crimes evidence
as part of its case-in-chief. MIMIC evidence is not dependent on defendant’s
introduction of favorable character evidence.

HYPO 19. Defendant is charged with the murder of Officer Johnson. The prosecution seeks to
prove that Defendant was convicted and imprisoned five years ago for narcotics sales in the
aftermath of an investigation and arrest made by Officer Johnson. Defendant objects on the
ground of impermissible character evidence. What ruling?

overruled. The prior narcotics crime tend to show the


defendant’s motive to kill Johnson.
EVIDENCE 21.

HYPO 20. Defendant is charged with possession of narcotics with the intent to sell. He defends
on the ground that he was merely a possessor and user—not a seller—of the drugs. The
prosecution seeks to prove that Defendant sold drugs a year ago in the vicinity of the arrest in the
current case. Admissible?
Yes, the defendant state of mind is in issue. Prior sale
shows his intent to do the same thing.

HYPO 21. Lizzie Borden is accused of intentionally killing her mother with an ax. Defense:
accident. Prosecution seeks to show that Lizzie threw a knife at her mother during a family
quarrel one week before the mother’s demise. The evidence:

(A) Is admissible because it shows Lizzie’s propensity for violence.


none of MIMIC can be used to show propensity
(B) Is admissible because it shows the ax incident was not an accident.

HYPO 22. D is charged with the armed robbery of a Wal-Mart in Indianapolis early in the
afternoon of July 1. Defense: mistaken identity. Prosecution seeks to introduce evidence that
around noon on July 1, D robbed a Penney’s and a Sears in Indianapolis, in the same vicinity as
the Wal-Mart.
Admissible, closeness in time and place of the other
crime. That tends to identify D as the robbery.

HYPO 23. Defendant is prosecuted for robbing the First National Bank. Defense: alibi.
Prosecution introduces evidence that the robber wore a red ski mask, carried a .38 caliber gun
and used a uniquely worded stick-up note. Prosecution then seeks to prove that Defendant used
the same modus operandi when robbing the Second National Bank a year ago.
If the MO is the other occasion is distinctive, and the
crime currently charged perpetrated in the same way.

Identity
22. EVIDENCE

HYPO 24. Defendant is charged with robbing the First National Bank. The prosecution seeks
to prove that two days before the robbery, Defendant stole a white Acura from a neighbor in the
same town. The robber of the First National Bank used a white Acura for the “getaway.”

Admissible. Common scheme or plan to rob the bank. In the


defendant mind, the seemingly unrelated crime are all part
of overall transaction to achieve a specific goal.

Method of proof of MIMIC-purpose crimes:

by conviction, or

by evidence (witnesses, etc.) that proves the crime occurred: conditional


relevancy standard—prosecution need only produce sufficient evidence
from which a reasonable juror could conclude that defendant committed
the other crime.
conditional relevancy evidence:P only needs to
provide sufficient evid that a reasonable
juror can conclude the D committed the crime
Upon defendant’s request, prosecution must give pretrial notice of intent
to introduce MIMIC evidence. In all cases, court must also weigh
probative value vs. prejudice and give limiting instructions if MIMIC
evidence is admitted.

MIMIC IN CIVIL CASES: If relevant to a non-character purpose,


MIMIC evidence can also be used in civil cases, such as tort actions for
fraud or assault.

F. Other Sexual Misconduct to Show Propensity in Sex-Crime Prosecution or Civil


Action
the defendant's prior specific acts of sexual assault
In a case alleging sexual assault, _________________________________________
______________________ ADMISSIBLE as part of the case-in-chief of the
prosecution (in a criminal case) or of the plaintiff (in a civil action) for the purpose of
the defendant's propensity
showing _________________________________________. In a case of child
molestation, same rule allows prior acts of child molestation.

Note: This rule allows prior acts only; not reputation or opinion.
not to be convictions
EVIDENCE 23.

II. AUTHENTICATION OF WRITINGS

Q-TIP: Whenever a writing appears on the exam, be alert to three potential issues (aside
from relevance): authentication, best evidence rule, and hearsay.

OVERVIEW: If the relevance of a writing depends upon its source or authorship,


a showing must be made that the writing is authentic (genuine), i.e., that it is what
it purports to be. This is the process of authentication.

In the absence of a stipulation as to authenticity, a FOUNDATION must be made


in order for the document to be admissible.

A. Methods of Authentication, In General

Issue: whether X is the author of DOCUMENT.

1. Witness’ personal knowledge: Witness observed X sign document.

2. Proof of Handwriting
Lay person’s opinion
(a) ___________________________ (Lay witness testifies to opinion that
X wrote document on basis of familiarity with X’s handwriting as
result of experience in normal course of affairs.)

authenticity is a Expert comparison opinion


preliminary issue
decided by jury (b) ___________________________ (Handwriting expert testifies to
opinion that X wrote document on basis of comparison between
document and genuine sample (exemplar) of X’s handwriting.)

Jury comparison
(c) ___________________________ (Jury compares document with
exemplar of X’s handwriting.)

3. Proof by Circumstantial Evidence

A party may rely on circumstantial evidence (anything relevant that connects


the alleged author to the document), such as appearance, contents, substance,
internal patterns or other distinctive characteristics (e.g., if the document
refers to information that only X would know, this may be used as an
inference that X is the author).
24. EVIDENCE

4. Ancient Document Rule—authenticity may be inferred IF document is:


At least 20 years old
(a) _________________________
Facially free of suspicion
(b) _________________________
Found in a place of natural
custody
(c) _________________________

5. Solicited Reply Doctrine

Document can be authenticated by evidence that it was received in


response to a prior communication to the alleged author.

Example: P e-mails contract offer to X, and later receives an acceptance


purportedly sent by X.

HYPO 25. During plaintiff’s case-in-chief, Witness testifies that, in her opinion, document was
written by X because she is familiar with X’s handwriting. X advises the judge that he intends to
testify during the defense that the document is a forgery and argues that the judge cannot admit
the document into evidence until the judge is personally convinced that the document was
written by X. Good argument?
No, when the question of fact to be raised

Conditional relevancy standard—document is admissible if court


determines there is sufficient evidence from which a reasonable juror
could conclude document is genuine, i.e., that X is the author.
the judge herself does not need to be convinced

some evidence in the record that the jury will


find
EVIDENCE 25.

B. Self-Authenticating Documents

In General. Presumed authentic—no need for foundation testimony:

(1) Official publications

(2) Certified copies of public or private records on file in public office

(3) Newspapers or periodicals

(4) Trade inscriptions and labels

(5) Acknowledged document


certification by a notary that the author appeared
before the notary and acknowledged to the notary that
she is in fact the author
(6) Commercial paper
promissory notes

C. Authentication of Photographs

Witness may testify on the basis of personal knowledge that the


fair and accurate presentation
photograph is a ______________________________________________
of the people or objects portrayed.

HYPO 26. Alice testifies that she observed the auto accident that occurred at the intersection of
Hickory and Elm Streets on July 1. She is shown a photograph and asked whether it is a fair and
accurate portrayal of the Hickory and Elm intersection as she remembers it on July 1.
“Objection: No foundation that Alice was the photographer.” What ruling?
Objection overruled.
It is sufficient the witness has the personal knowledge.
26. EVIDENCE

III. BEST EVIDENCE RULE


writing
Definition: A party who seeks to prove the contents of a ______________ must either
produce the original writing an acceptable excuse for absence
_________________________________ or provide ____________________________.

If the court finds the excuse is acceptable, the party may then use secondary evidence—
oral testimony or a copy.

NOTE: The definition of writing includes sound recordings, X-rays, and films.

HYPO 27. Bubba ordered 100 pounds of shrimp from Gulf Shrimp Co. pursuant to a written
purchase order. In his suit for breach of contract, Bubba takes the stand and testifies, “I didn’t
get what I ordered. The purchase order called for 3” jumbo shrimp and they delivered 1” mini-
shrimp.” Which of the following would be a valid objection to Bubba’s testimony?

(A) The actual shrimp are the best evidence of what was delivered.

(B) The purchase order is the best evidence of what the contract required.

A. When best evidence rule applies: When a party is seeking to prove the
contents of a writing.

Two principal situations:

1. The writing is a legally operative document, i.e., the writing itself creates
rights and obligations. Examples: patent, deed, mortgage, divorce decree,
written contract.

2. Witness is testifying to facts that she learned solely from reading about them
in a writing.
EVIDENCE 27.

HYPO 28. Tommy the Terrorist is charged with detonating a bomb. No one witnessed the
detonation, but it was captured on film by an unmanned surveillance camera. Counterterrorism
Agent Jack Bower testifies that he watched the film and it clearly shows Tommy was the
bomber. Objectionable?
Yes. It violates the best evidence rule: even though the film is
not the legal operated doc, Jack Bower does not have personal
knowledge about the bombing by reference to the content of the
film. Jack is trying to prove the content of the writing. Show the
jury film.

B. When best evidence rule does not apply: When a witness with personal
knowledge testifies to a fact that exists independently of a writing that records
the fact.

HYPO 29. Agent Jack Bower is prosecuted for giving perjured testimony at a congressional
hearing into the use of torture during the interrogation of terrorist suspects. At trial, a
congressional aide offers to testify to what Jack said during the hearing. True or False: The
aide’s testimony is improper because the transcript is the best evidence of what Jack said.
False. Best evidence rule does not apply.
1. transcript is not legally operative doc
2. in the fact there is a witness who has personal knowledge of
what Jack said.

HYPO 30. Agent Jack Bower, claiming he worked a 24-hour shift, sues Boss for nonpayment of
wages and failure to reimburse for expenses.

(a) Without producing any documents, Jack testifies, “I worked 24 hours and my
expenses were $15 million.” Boss objects—“Best evidence rule. Produce the time
sheets and expense receipts.”
overruled. Best evidence rule does not apply here.
1. time sheets and expense receipts are not legally operative doc
The time sheets do not create hours that Jack worked. He worked
those hours regardless of the time sheets.
2. Jack has the personal knowledge.

(b) Without producing any documents, Boss testifies: “Jack’s time sheets show he
worked only 20 hours, and the receipts show only $10 million in expenses.”
Boss’ testimony does violate the best evidence rule. Because Boss
has no personal knowledge. His sole knowledge comes from reading
the sheets and receipts.
28. EVIDENCE

C. What qualifies as the “original writing”?

1. Whatever the parties intended as the original; any counterpart intended to


have the same effect; any negative of film or print from the negative;
computer print-out.

2. duplicate—any counterpart produced by any mechanical means that


accurately reproduced the original (e.g., photocopy, carbon copy). Rule on
duplicates: duplicate is admissible to same extent as original UNLESS it
unfair
would be _____________ (e.g., photocopy of fuzzy fax), or genuine question
is raised as to authenticity of original.

3. handwritten copy is neither an original nor duplicate.

D. Excuses for non-production of original

1. lost or cannot be found with due diligence

2. destroyed without bad faith

3. cannot be obtained with legal process


no subpoena power

Court must be persuaded by preponderance of the evidence that excuse has been
established; secondary evidence is then admissible (e.g., testimony based on
memory, handwritten copy).

E. “Escapes”

Voluminous records
1. _______________________ can be presented through a summary or chart,
provided the original records would be admissible and they are available for
inspection.

public records
2. Certified copies of ____________________________

Collateral documents
3. ____________________________________

If court, in its discretion, determines writing is collateral, contents may be proven


by secondary evidence.
EVIDENCE 29.

IV. WITNESSES

A. Competency of Witness, In General


(the testimonial qualifications a witness must possess in order to be allowed to testify)

1. Basics:

(a) Personal Knowledge


she saw with her own eyes or heard with her own ears

(b) Oath or Affirmation


witness must demonstrate the willingness to tell the
truth

B. “Dead Man’s Statute”

In General (Multistate rules)

(1) Witness is not ordinarily incompetent merely because she has an


interest—a direct legal stake—in outcome of the litigation.

(2) BUT some states have a “Dead Man’s Act.” The typical statute
provides:
CIVIL
i. in a _________________ action,
interested
ii. an ___________________ witness
incompetent
iii. is ___________________ to testify
decedent
iv. against the estate of a ______________
a transaction or communication
v. concerning ______________________________
between the interested witness and the decedent.
30. EVIDENCE

HYPO 31. Shania sued Elvis for breach of an oral contract. Elvis denied that any contract was
made. Elvis died before trial. (a) May Shania testify to what Elvis said and did in negotiating
the contract? (b) May Shania’s friend Faith, who witnessed the making of the contract, testify to
what Elvis said and did?
YES
Yes
Shania and Faith are both competent to testify what Elvis said and did

Under the FRE, there is no “dead man’s rule.” Thus, on Multistate exam,
witnesses ordinarily are not incompetent on this ground. BUT, if question
explicitly states that the particular jurisdiction in which the case arises has
a “dead man’s statute,” apply the rule in B.(2) above.
(a) Shania is an interested witness(plaintiff),
therefore now Shania becomes incompetent to testify what
Elvis said and did.
(b)Faith. Faith has no direct legal stake in the
outcome, therefore, Faith is competent. Even though
Faith is a bias witness, bias is not a disqualifying
interest and she can testify.

C. Leading Questions

Form of question suggests the answer (e.g., “Isn’t it a fact that you ran the red light?”; or
unevenly balanced alternatives, such as “Were you driving fast and furious, or in some
other way?”).
leading questions are not
allowed
(a) GENERALLY ________________________ on DIRECT EXAMINATION
of witness.
leading questions
are allowed
(b) Generally ________________ on CROSS-EXAMINATION of witness.

(c) BUT allowed on DIRECT EXAM as follows:


Preliminary introductory matters
(1)_____________________________
A youthful or forgetful witness
(2)_____________________________
A hostile witness
(3)_____________________________
Opposing party, someone under
control of opposing party
(4)_____________________________
EVIDENCE 31.

D. Writings in Aid of Oral Testimony

1. Refreshing Recollection

(a) Basic rule: Witness may not read from prepared memorandum; must
testify on basis of current recollection.

(b) BUT if witness’s memory fails him, he may be shown a memorandum


(or any other tangible item) to jog his memory.

HYPO 32. Homer Simpson’s house was burglarized two years ago, and several valuable items
were stolen. Simpson sued his insurer for failing to pay the loss covered by his homeowner’s
policy. While on the stand at trial, Homer has trouble remembering all of the stolen items. To
refresh Homer’s recollection, his attorney shows him a copy of a list of the missing items that
Homer prepared for the police the day after the burglary. Insurer objects on the ground of lack
of authentication, best evidence rule and hearsay.

(a) What ruling?


overruled. You can show literally any writing to try to refresh
witness recollection. There is no need for authentication. Because
the writing is not being offered as the evidence.

(b) If Homer’s recollection is refreshed, may he then read the list into evidence?
No. Not allowed. After the witness’ memory has been restored, the
writing must be set aside.

(c) Safeguards against abuse: adversary has right:

(1) to inspect the memory-refresher

(2) to use it on cross-examination

(3) to introduce it into evidence


32. EVIDENCE

2. Past Recollection Recorded (Hearsay Exception)

HYPO 33. In Hypo 32, Homer looks at the list of stolen items he prepared for the police the day
after the burglary. It fails to jog his memory, and he is still unable to testify on the basis of
current recollection. At this point, Homer’s attorney seeks to read the list into evidence.
Objection: hearsay.

Hearsay exception for past recollection recorded. Foundation for reading


the contents of the writing into evidence:
allow the writing to be read to the jury, substitute for
the witness’ lack of memory
(1) showing writing to witness fails to jog memory

(2) witness had personal knowledge at former time

(3) writing was either made by witness, or adopted by witness

(4) making or adoption occurred _________________________


while the event is still fresh in the witness’ memory
_________________________________________________

(5) witness can vouch for accuracy of writing when made or


adopted.

HYPO 34. After laying foundation, Homer’s attorney seeks to introduce Homer’s memorandum
into evidence as an exhibit.

(a) Proper?
No, the memo can now be read to the jury, but the memo itself cannot
be introduced by the party who is using the memo as the substitute
for the witness memory.
Don’t want to give too much weight.

(b) May the insurer have the memorandum introduced as an exhibit?


If the opponent wants to put the evidence in exhibit, the opponent
has that option, if the opponent feels for tactic reasons.
The option is only for the opponent party.
EVIDENCE 33.

E. Opinion Testimony

1. Lay Witness

(a) Lay opinion admissible if:


the opinion is rationally based on witness’ own
perception
(1) _______________________________________
(personal knowledge), and
the opinion must be helpful to the jury in deciding
the case
(2) _______________________________________

(b) Examples:

drunk/sober

speed of vehicle

sane/insane

emotions of another person

odors

handwriting

character (when permitted)

2. Expert Witness

(a) Qualifications:

education

AND/OR experience

(b) Proper Subject Matter

Scientific, technical or other specialized knowledge that will be


helpful
__________________ to jury in deciding a fact. E.g., an opinion

is not helpful if the proposition is obvious.


34. EVIDENCE

(c) Basis of Opinion

Expert must have opinion based on “reasonable degree of


probability or reasonable certainty”

The expert may draw upon three permissible data sources:


Personal knowledge
(1) _________________________ (e.g., treating physician)
Other evidence in the trial record
(2) _____________________________________ (testimony by
other witnesses, exhibits (medical reports, X-rays))—made
known to expert by hypothetical question

(3) facts outside the record (hearsay) if ____________________


this outside material is of a type reasonablt relied upon by expert
in a particular field in forming opinions.
_________________________________________________

(In general, the contents of such out-of-court material should


not be disclosed to the jury unless the judge determines it
would help the jury evaluate the expert’s opinion. If the judge
makes such a determination, the out-of-court material may be
admitted with a warning to the jury to consider it only in
evaluating the quality of the expert’s opinion.)

HYPO 35. Dr. Seuss, a board-certified child psychiatrist, testifies, “In my opinion, within a
reasonable degree of medical probability, Bartholomew Cubbins’ preoccupation with hats is a
disabling psychosis. My opinion is based on (1) my own clinical interviews and tests of
Bartholomew; (2) exhibits 1 and 2 in evidence (MRI test results, medical office records of Dr.
Grinch); (3) interviews of Bartholomew’s friends Wump, Gump and Thump; and (4) a written
report prepared by Dr. Sam I. Am. As to items (3) and (4), this is the type of information that
psychiatrists like me customarily rely upon in making evaluations.”

(a) Bartholomew moves to strike Dr. Seuss’s opinion because it is based, in part, on
inadmissible hearsay.
Item 3,4, the psychiatrist reasonably relied upon in forming clinical
opinion

(b) Should Dr. Seuss be permitted to testify further, “Let me tell you what Wump said
during our interview, and let me read you what was in Dr. Sam I. Am’s report”?
Generally the testimony in item B goes too far.
Hearsay danger.
Jury will use that as the substance evidence-hearsay sneaks into
the back door.
If the judge decides that it would be help to the jury to
evaluate the experts credibility.
EVIDENCE 35.

(d) Relevance and Reliability


The expert must use
reliable methods and To be admissible, expert opinion must be relevant to the issue at
principles, and apply
them reliably to the hand and SUFFICIENTLY RELIABLE.
facts of the case
Court serves as “gatekeeper,” and will use four principal factors to
determine reliability of principles and methodology used by expert
(all types) to reach opinion (Daubert)—“TRAP”
1. Testing: has there been testing principle or methods?
2. Rate of error: high or low
______________________________
3. Acceptance: acceptance by other experts in the same discipline
(Note: general acceptance is not required, you don't need to have
consensus. It's a factor)
4. Peer review and publication.
______________________________

______________________________

______________________________

(e) Learned Treatise in Aid of Expert Testimony (Hearsay Exception)

(1) On direct examination of party’s own expert:


cannot use the treatise by yourself, need to be with experts

Relevant portions of treatise, periodical, or pamphlet may be


read into evidence as substantive evidence (to prove truth of
matter asserted) if established as reliable authority.

(2) On cross-examination of opponent’s expert:

Read into evidence to impeach and contradict opponent’s


expert. Comes in as substantive evidence.

(3) BUT learned treatise may not be introduced as exhibit.


when jury see sth in print, they give too much
weight
36. EVIDENCE

3. Ultimate Issues

Opinion testimony (lay or expert) is not objectionable just because it


embraces an “ultimate issue” in the case (e.g., in DWI case, layperson
testifies “X seemed drunk” could be admitted). BUT, all other
requirements for opinion testimony must be satisfied, including the
helpful to the jury
requirement that the opinion is ________________________ (e.g.,
witness would not be allowed to give opinion that “defendant is guilty (or
innocent)”).

HYPO 36. In a personal injury case, Defendant is alleged to have been driving recklessly at the
time of a car accident. Witness who observed the event testifies that Defendant looked angry,
smelled of alcohol and drove away from the scene at 80 m.p.h. Witness then states, “It looked to
me as though Defendant was engaged in conduct constituting a reckless disregard for the safety
legal jargon
of others.” Objectionable?

(A) Yes, because Witness is testifying to the ultimate issue. objection has been abolished.

(B) Yes, because Witness’s opinion is not helpful. to lay person in jury

Criminal Cases: “Ultimate issue” is still proper objection if expert seeks


to give direct opinion that defendant did or did not have relevant mental
state. Thus, the following is not allowed: “D’s insanity prevented him
from understanding that he was shooting at the victim in this case.” The
expert can only testify in general terms about the effects of a defendant’s
mental condition without linking it to the particular case, such as: “D has
schizophrenia. A person with such disease cannot distinguish fact from
fantasy.”

F. Cross-Examination

1. Party has a RIGHT to cross-examine any opposing witness who testifies at the
trial. Significant impairment of this right will result, at minimum, in striking
of witness’s testimony.
EVIDENCE 37.

2. Proper subject matter:


within the scope
(a) Matters ________________________ of direct examination.

credibility
(b) Matters that test the witness’s __________________

G. Credibility and Impeachment, In General

1. Bolstering Own Witness


after the witness’ credibility has
been attacked
The rule: not allowed until ________________________

(Post-impeachment repair of witness’s credibility is called rehabilitation).

HYPO 37. Plaintiff calls Witness 1 to the stand. Witness 1 testifies that she saw Defendant’s car
run the red light. Defense counsel states that she has no questions for the witness. After Witness
1 steps down, Plaintiff calls Witness 2 who testifies, “Witness 1 has a good reputation for
truthfulness.” Objectionable?
Yes. Objection sustained. Improper bolstering.
no attack on witness credibility

Variation: Witness 1, after testifying that she saw Defendant’s car run the red light, then
testified, “I told everyone at work the next day that I had seen Defendant run the red light.” This
is an inadmissible prior consistent statement:

improper bolstering.
1. No attack
2. Prior consistent statement only has
limited probative value
3. Hearsay
38. EVIDENCE
Witness prior identification of a
EXCEPTION: person
____________________________ (e.g., Witness testifies
that she recognizes D, sitting in court, as the perpetrator. In addition, “I
picked D out of a line-up two weeks after the robbery”). Might seem like
hearsay (out-of-court statement offered to prove truth of statement) but
prior identification by trial witness is not barred by hearsay rule. It is
labeled as “exclusion” from hearsay, and comes in as substantive
evidence.

NOTE: Witness who made prior identification must testify at trial and
must be subject to current cross-examination.

2. Impeachment of Own Witness


it is permitted without limitation
The rule: ________________________.

Note: Even though the Federal Rules speak only of impeaching a


witness on cross-examination, a party can impeach her own witness
during direct examination.

H. Impeachment Methods

Overview of Methods:

(1) Prior Inconsistent Statements


(2) Bias, Interest or Motive to Misrepresent
(3) Sensory Deficiencies
(4) Bad Reputation or Opinion about witness’s character for truthfulness
(5) Criminal Convictions
(6) Bad Acts (without conviction) that reflect adversely on witness’s
character for truthfulness
(7) Contradiction

Procedure Overview: There are two possible ways to use impeachment methods:

(1) Ask the witness about the impeaching fact with the aim of having
the witness admit it (“confronting” the witness), or

(2) Prove the impeaching fact with “extrinsic” evidence (documentary


evidence or testimony from other witnesses).

(1) The impeaching fact may be proven with extrinsic evidence as to the
following impeachment methods:
All of them except 6.BAD ACTS, 7.contradictory facts that are
collateral.
EVIDENCE 39.

(2) For the impeachment methods that allow extrinsic evidence, is it necessary
to ask the witness about the impeaching fact before the extrinsic evidence is
introduced?
No, except for 2.Bias

1. Prior Inconsistent Statements

Any witness may be impeached by showing that on some prior occasion,


she made a material statement (orally or in writing) that is inconsistent
with her trial testimony.

GENERAL PURPOSE: The prior inconsistent statement is admissible


only for the purpose of
impeachment
__________________ (to suggest trial testimony is false or mistaken, not
as substantive evidence that the statement is actually true).
cannot use of substantive evid (to prove the truth of the
matter)
EXCEPTION: A prior inconsistent statement of a witness may be
admitted both to impeach and as substantive evidence (to prove the truth
of the matter asserted in the prior statement), if the witness is currently
subject to cross-examination and the prior inconsistent statement was
made:
orally under oath
(a) _______________________________________, and
as a part of formal hearing, proceeding, trial or
(b) ___________________________________________
deposititon

___________________________________________
(hearsay exclusion)

HYPO 38. Defendant is sued for negligence in a multi-vehicle accident in which he was driving
his Suburban. Witness testifies for plaintiff that she saw the Suburban run the stop sign.

(a) On cross-examination, may Defendant’s counsel seek to establish that a few days
after the accident, Witness told the police that the Jeep Cherokee, not the Suburban,
ran the stop sign?
inconsistent prior statement

(b) If Witness admits she made the prior inconsistent statement, may Defendant use the
statement as substantive evidence that the Jeep Cherokee, rather than the Suburban,
ran the stop sign?
No.
Prior inconsistent statement is hearsay offered
as substantive evidence.
40. EVIDENCE

(c) What if Witness made her prior inconsistent statement about the Jeep Cherokee
during a pretrial deposition in which she gave sworn testimony?
The prior inconsistent statement would be admissible for 2
purposes. Impeach & substantive evidence.

before grand jury is ok

Procedural Issue: Must Witness be confronted with her prior inconsistent


statement while still on the stand, or may it be proven later by extrinsic evidence
without such confrontation?

Rule: Confrontation timing is flexible: Not required to immediately confront


Witness. But after proof by extrinsic evidence, Witness must be given an
opportunity at some point to return to stand to explain or deny the prior
inconsistent statement.

Exception: No opportunity to explain need be given if Witness is


the opposing party
___________________. Also, the prior inconsistent statement of an opposing
party can be used against that party as substantive evidence (“party admission” or
“statement of an opposing party”).

HYPO 39. In auto accident case, Plaintiff testifies that she was wearing her seat belt. Defendant
does not cross-examine her. During the defense, Defendant calls Joe the Bartender, who testifies
that Plaintiff told him, at Joe’s bar a week after the accident, that she had NOT been wearing her
seat belt.

(a) Should Plaintiff’s motion to strike be granted on the ground that Plaintiff was not
given an immediate opportunity to explain or deny the inconsistency?
No, there is no need to give any opportunity to this witness to
explain or deny. Because the witness happened to the opposing
party.

(b) Is Plaintiff’s statement admissible to impeach Plaintiff AND as substantive evidence


that she was not wearing her seat belt?
Yes. Even though this prior inconsistent statement was made
informally, this statement is made by opposing party, and it
can be used as substantive evidence(party admission)
Note: pay close attention to the status of the witness
EVIDENCE 41.

2. Bias, Interest, or Motive to Misrepresent

Bias may be based on any fact that would give a witness a reason to testify
favorably or negatively about a party’s case. Examples: Witness is party;
friend, relative or employee of party; expert witness being paid by party;
person with grudge against a party, etc. Purpose: to suggest testimony is
false, slanted, or mistaken in party’s favor.
A prosecution witness who is testifying pursuant to a plea
bargain
Procedural Issue:

(a) Must witness be confronted with alleged bias while on the


stand?
Yes.

(b) If confrontation prerequisite is met, may bias be proven by


extrinsic evidence?
Yes.
.

3. Sensory Deficiencies

Anything that could affect witness’s perception or memory. Examples:


bad eyesight, bad hearing, mental retardation, consumption of alcohol or
drugs at time of event or while on the witness stand. Purpose: to suggest
mistake.
No
Confrontation required? _______

Yes
Extrinsic evidence allowed? _______
42. EVIDENCE

4. Bad Reputation or Opinion About Witness’s Character for Truthfulness

Any witness is subject to impeachment by this method.

No
Confrontation required? ________
Yes
Extrinsic evidence allowed? _______

Call a character witness to testify that Target Witness has bad reputation
for truthfulness, or that character witness has low opinion of Target
Witness’s character for truthfulness. Purpose: to suggest that Target
Witness is not telling the truth on the witness stand.

HYPO 40. Larry testifies for the prosecution that he saw Defendant commit the crime. During
the defense: Defendant calls Rev. Al to testify that Larry has a lousy reputation for truthfulness
among members of Rev. Al’s congregation, and in Rev. Al’s opinion, Larry is not a truthful
person.

(a) Admissible to suggest Larry’s testimony is false?


Yes.

(b) May Rev. Al follow up his opinion as follows: “Let me tell you how I reached my
opinion of Larry. During the past year, he lied to me on six separate occasions.”
No. That’s evidence of specific acts.
Character witness cannot be allowed to prove some other bad
character for truthfulness by using specific acts.
not even to show his basis of opinion
Allowed: reputation & opinion

5. Criminal Convictions

Purpose: to suggest testimony is false. Like all impeachment methods,


this is allowed in both civil and criminal cases. Relevance: person who
has been convicted of a crime is more likely to lie under oath than is a
person with an unblemished record.
EVIDENCE 43.

Permissible Types of Convictions:


The ct has no
discretion to exclude (1) Conviction of any crime (felony or misdemeanor) as to which
this type of
conviction.Interpreted the prosecution was required to prove:
narrowly, truth- false statement as an element of the crime
telling type of crime: _________________________________________
the uttering of false
words(orally & in
writing) eg: perjury,
fraud, not mere theft
(2) If conviction did not require proof of false statement, it must be
felony
a ___________, and court may exclude, in its discretion, if
probative value on issue of witness credibility is outweighed by
danger of unfair prejudice to a party (misuse as evidence of
liability or guilt). Factors that make a felony conviction
probative on credibility: seriousness of crime; relation to
deception and stealth. Prejudice factors: similarity to case
being tried; inflammatory nature of prior crime.

Time Limitation:

As to both categories, the conviction, or release from prison,


whichever is later, generally must be within 10 years of trial. If
more than 10 years have elapsed, the conviction may not be used
for impeachment UNLESS:
the proponent shows that the probative value on
credibility is substantial
HYPO 41. Defendant is prosecuted for arson. At trial, Defendant testifies in his own behalf,
urging that the fire was an accident. On cross-examination, may the prosecutor properly ask
Defendant:

(a) whether he was convicted eight years ago for the misdemeanor of income tax fraud?
Yes. Conviction involving false statement

This can be used to impeach any witness including the criminal


defendant if the defendant testifies.
(b) whether he was released from prison a year ago for his misdemeanor conviction for
possession of marijuana?
No, not a crime of false statement, not a felony

(c) whether he was convicted two years ago for the misdemeanor of shoplifting?
No, not felony, not a crime of false statement.
Don’t need to say anything to commit the crime, no uttering of
false words.
44. EVIDENCE

(d) whether he was released from prison three years ago for a murder conviction?
Yes, in the ct’s discretion.

Method of proof:

Ask witness to admit prior conviction, or

Introduce record of conviction (extrinsic). Not required to


confront witness prior to introduction of record of
conviction.

6. Inquiry About Bad Acts (without conviction) if they reflect adversely on


witness’s character for truthfulness
bad act of deceit or lying To confront witness on cross
examination and hope witness admits
The only permissible procedure: ______________________________
No extrinsic evidence is allowed

Cross-examiner must have good-faith basis for the inquiry, and permission
to make the inquiry is subject to the court’s discretion. The inquiry is
limited to the act of untruthfulness itself, not its consequences, such as job
termination, civil judgment, or arrest.

Exam Tip: Proof with extrinsic evidence may still be allowed if the bad
act is relevant for some purpose other than bad character for truthfulness.
Such as bias
HYPO 42. Witness gives favorable testimony for Defendant. On cross-examination, Plaintiff
asks Witness whether she assaulted her mail carrier two years ago (no charges were brought).
No, assault is not a bad act reflecting character for untruthfulness

HYPO 43. After Witness testifies for Defendant, Plaintiff asks Witness whether she made false
statements in an application for food stamps in July 2001 (no charges were ever brought).
This inquiry is allow in ct’s discretion
If the witness admits, it casts some doubt on her current credibility
EVIDENCE 45.

HYPO 44. Same cross-examination. Witness vehemently denies making false statements in the
application for food stamps. May Plaintiff thereafter call a welfare agent to prove that Witness
made the false statements?
No, no extrinsic evidence is allowed.

HYPO 45. Prosecution of Michael for embezzlement of the office petty-cash fund. Dwight
testifies for Michael. On cross-examination, Dwight is asked whether he was arrested three
years ago for passing counterfeit money. Objectionable?
Yes, objectionable. You are not allowed to ask the witness if the witness
himself was arrested for his bad act.

HYPO 46. Prosecution of Donald. Winston testifies for the prosecution. On cross-examination,
Winston is asked whether he was arrested a month ago for selling marijuana and is awaiting trial
on those charges.

Allowed. Winston’s arrest situation is a source of bias.


If Winston denies, extrinsic evidence can be used to prove bias.

7. Contradiction

Concept: Cross-examiner, through confrontation of witness, may try to


obtain admission that she made a mistake or lied about any fact she
testified to during direct examination. If the witness admits the mistake or
lie, she has been impeached by contradiction. However, if she sticks to
her story, the issue becomes whether extrinsic evidence may be introduced
to prove the contradictory fact.

Rule: EXTRINSIC EVIDENCE NOT ALLOWED for the purpose of


collateral
contradiction IF the fact at issue is ____________ (i.e., if the fact has no
significant relevance to the case or to the witness’s credibility).
46. EVIDENCE

HYPO 47. In an auto accident case, Witness testifies for Plaintiff that, while leaning against a
maple tree near the intersection of Boardwalk and Park Place on March 1, he saw that the traffic
light was red for Defendant as Defendant’s car entered the intersection and hit Plaintiff. On
cross-examination, Witness is asked (a) “Isn’t it a fact that the tree at that intersection is an oak?”
and (b) “Isn’t it a fact that the traffic light was not functioning at all on March 1?” Witness
insists that his direct testimony was accurate.

(a) During the defense, may Defendant properly prove that the tree at the intersection is
an oak tree?
No, collateral contradictory fact

(b) During the defense, may Defendant properly call a police officer to testify that the
traffic light at the intersection was not functioning at all on March 1?
Yes. Non-collateral contradictory fact.

I. Rehabilitation

1. Showing witness’s good character for truthfulness.

(a) When?
Only when the impeachment suggests the witness was
lying as compared to merely being mistake.

(b) How?
Bring out character witness to testify that witness
has good character for truthfulness, even though
reputation testimony or opinion testimony.

2. Prior consistent statement to rebut.

(a) To rebut a charge of recent fabrication:

When? If the witness’s trial testimony is charged as a recent


fabrication (e.g., product of recent improper influence or bias), a
prior statement by the witness that is consistent with her testimony
will be admissible to rebut the charge IF the statement was MADE:
before the motive to fabricate arose
_________________________________________
EVIDENCE 47.

(b) To rebut a contention of inconsistency:

Example: A police officer who testifies that she singled out the
defendant because of a prominent scar on his face is impeached
with a showing that the officer omitted to mention the scar in her
informal handwritten report. For rebuttal, it could be shown that
the officer mentioned the scar in her formal typewritten report.

(c) To rebut a contention of sensory deficiency:

Example: A witness is impeached with a contention of faulty


memory based on the fact that a year after the accident about
which she testified, she underwent treatment for a mental disorder.
For rebuttal, it could be shown that she gave the same account of
the accident to the police shortly after the accident happened.

Purpose: A prior consistent statement that fits within any of the


rebuttal rules is admissible to rehabilitate credibility and as
substantive evidence that the prior statement was true. It is labeled
a hearsay exclusion.

HYPO 48. Brad v. Jennifer. On July 1, pedestrian Brad was struck by a car driven by Jennifer.
Angelina, a stranger to Brad and Jennifer at the time, witnessed the accident and told the police
on July 1 that Brad looked sober as he crossed the street. At trial, six months later, Angelina
testifies for Brad, “He looked sober as he crossed the street.”

(a) On cross-examination, the only question Angelina is asked is whether she was
convicted eight years ago of income tax evasion, to which she answers “Yes.” On re-
direct, may Angelina properly testify that she told the police on July 1 that Brad had
looked sober?
No.There is no basis here for using consistent statement.
That is a general attack on bad character for truthfulness, not a
charge for recent fabrication.

(b) Assume that on the cross-examination of Angelina, she is asked, “Isn’t it a fact that
after this accident, you and Brad became close friends and are now living together as
lovers?” to which she answers, “Yes.” On re-direct, may Angelina properly testify
that she told the police on July 1 that Brad had looked sober? If so, for what purpose?
yes, now we have the insinuation that Angelina testifies in favor of
Brad because of the motive and influence of their romance.

She said it before the romance. That prior consistent statement made
before the motive to fabricate. It neutralize the influence of
romance.
It’s admissible as additional substantive evidence that Brad was in
fact sober.
48. EVIDENCE

V. PRIVILEGES

A. Introduction

1. In general, on MULTISTATE exam, apply basic rules on privileges as


covered in lecture.

2. Federal procedure issue on MULTISTATE exam: If bar examiners


specifically indicate the action is pending in federal court, apply the following
procedural rules:

(a) In a federal-court action ARISING UNDER FEDERAL


SUBSTANTIVE LAW (all civil cases arising under Constitution or
federal statutes, and all criminal cases): “privileges are governed by
the principles of the common law as they may be interpreted by the
federal courts in the light of reason and experience.” For the most
part, these are the basic rules on privileges as covered in lecture.

(b) In a federal-court action based on DIVERSITY jurisdiction, where


state substantive law applies to parties’ claims and defenses (Erie
situation), the federal court must apply PRIVILEGE LAW OF THE
STATE whose substantive law is applicable.

Note: In diversity actions, federal courts also apply STATE LAW ON


COMPETENCY (e.g., Dead Man’s Statutes) and STATE LAW ON
BURDENS OF PROOF AND PRESUMPTIONS. Aside from these
three exceptions (privileges, competency, and burdens of proof /
presumptions), FRE apply in all federal-court actions, including
diversity cases.

B. Attorney-Client Privilege

1. Rationale: To encourage client to speak openly to counsel.

2. Elements. Privilege applies to—


1. confidential communication
2. between attorney and client, or the representative of either
_____________________________________
3. made during a legal professional consultation, unless the privilege waived by the
client
4. unless an exception applies
_____________________________________

_____________________________________

_____________________________________

_____________________________________
EVIDENCE 49.

3. Definitions:

(a) Confidential communications: Client must have reasonable


expectation of confidentiality (e.g., no privilege if client knows that
third party is listening in; or if client asks attorney to disclose the
communication to a third party). Joint client rule: If two or more
clients with common interest consult the same attorney, their
communications with counsel concerning the common interest are
privileged as to third parties. But if the joint clients later have dispute
with each other concerning the common interest, privilege does not
apply as between them.

Communication: Privilege covers the exchange of information


between attorney and client. It does not apply to the client’s
knowledge of the underlying information, pre-existing documents,
or physical evidence.

HYPO 49. Delbert is sued for his alleged negligence in an auto accident. He tells his attorney
what happened and gives her the cell phone with which he was making a call at the time of the
accident. Before trial, Delbert is deposed by plaintiff’s counsel:

(a) Must Delbert respond if asked, “What did you tell your attorney about the accident?”
No. Privilege communication

(b) Must Delbert respond if asked, “Describe what you were doing at the time of the
accident.”
Yes. Knowledge of underlying facts.

(c) If served with a subpoena, must Delbert’s attorney produce Delbert’s cell phone?
Yes. Cell phone is the physical evidence, not communication with
counsel

(b) Attorney—member of the bar or person that client reasonably believes


is member of the bar

Representative of the attorney—any agent reasonably necessary to


facilitate the provision of legal services (e.g., accountant working
with attorney to “translate” client’s financial matters)

(c) Client—includes person seeking to become client (e.g., privilege


attaches at outset of formal consultation with attorney even if client
does not retain attorney) including initial
communication
50. EVIDENCE

Representative of client—any agent reasonably necessary to


facilitate the provision of legal services (e.g., for corporate client,
any employee who communicates with corporation’s attorney to
enable attorney to provide legal services to the corporation)

(d) Professional legal consultation—primary purpose of communication


must be to obtain or render legal services, not business or social advice

(e) Waiver—
CLIENT
(1) Voluntary Waiver: Only the ________________________
has the power to waive the privilege. After the client’s death,
the privilege continues and only the client’s estate can waive it.

(2) Subject Matter Waiver: A voluntary waiver of the privilege


as to some communications will also waive the privilege as to
other communications if:
intentional
(a) The partial disclosure is ______________________,

(b) The disclosed and undisclosed communications


same subject matter
concern the ____________________________, and
fairness
(c) _________________ requires that the disclosed and
undisclosed communications be considered together.

(3) Inadvertent Waiver: An inadvertent disclosure of a


privileged communication will not waive the privilege so long
as the privilege-holder:
reasonable steps to prevent
(a) took _____________________________________
the disclosure, and
correct
(b) takes reasonable steps to ___________________the
error.
EVIDENCE 51.

(f) Exceptions—
Future crime or fraud
(1) ____________________________: E.g., client tells attorney,
“Help me disguise the bribes I made so that they look like
legitimate business expenses.”

Client puts legal advice in issue


(2) ____________________________: E.g., in tax fraud
prosecution, defendant defends on ground that she relied on
advice of her attorney in reporting income.

Attorney-client dispute
(3) ____________________________: E.g., attorney sues client
for unpaid fee, or client sues attorney for legal malpractice.

C. Physician-Patient Privilege
anyone who is professionally certified.
1. Usually created by state statute. Rationale: to encourage candor by patient
and to protect privacy.

2. Elements. Privilege applies to—

Confidential communication or information acquired by physician from


patient for purpose of diagnosis or treatment of medical condition

Also applicable to psychotherapists (M.D. or other professional certified


to diagnose or treat mental / emotional illness).

Federal law distinction: in federal-court actions based solely on federal


substantive law (e.g., civil rights, securities fraud, federal criminal case),
psychotherapy
privilege exists only for ______________________. (There is no privilege
in federal-court actions based solely on federal law for confidential
communications with physicians as regards physical conditions.)

HYPO 50. Physician examines Patient’s lungs in hospital room while visitor is present. (1)
Patient tells doctor, “Do you suppose my wheezing is due to the four packs of cigarettes I smoke
every day?” (2) After visitor leaves, Patient says to doctor, “Know any good lawyers? I haven’t
paid my income taxes in three years.”
52. EVIDENCE

(a) In state court action in which condition of patient’s lungs is an issue, could doctor be
compelled to disclose statement (1)?
Yes, even though it’s info related to treatment and diagnose, it was
not confidential—a visitor was present. No privilege.
(b) In prosecution for income tax evasion, could doctor be compelled to disclose
statement (2)?
Yes. It’s not related to treatment and diagnose.

3. Exception applicable to both physician and psychotherapist privileges: If


patient expressly or impliedly puts physical or mental condition in issue. E.g.,
patient is plaintiff suing for damages for personal injury, or defendant asserts
insanity defense.

D. Spousal Privileges

1. Spousal Immunity—Criminal Cases Only

Rule: A spouse cannot be compelled to testify about anything against the


defendant spouse. (Sometimes called “privilege against adverse spousal
testimony.”)

Rationale: to protect harmony of existing marriage at time of trial.


the witness spouse, not the defendant
Holder of the privilege: ____________________________________

(Witness-spouse may voluntarily testify against the defendant


spouse if he/she so chooses.)

2. Confidential Communications Between Spouses

IN ANY TYPE OF CASE, a spouse is not required, and is not allowed in


the absence of consent by the other spouse, to disclose a confidential
communication (statements or acts) made by one to the other during the
Both spouses, either can
marriage. Holder of this privilege: ______________________
invoke
Rationale: to encourage candor between husbands and wives during the
marriage.
EVIDENCE 53.

3. Exceptions applicable to both privileges:

communications or acts in furtherance of jointly-perpetrated future crime


or fraud

communications or acts destructive of family unit, e.g., spousal or child


abuse

in litigation between the spouses themselves (e.g., divorce, breach of


contract)

HYPO 51. Niles is prosecuted for the murder of his brother Frazier. Niles and Daphne are a
married couple. Niles comes home on the night of Frazier’s demise wearing a blood-stained
Armani topcoat, which Daphne observed.

(a) At trial, the prosecutor calls Daphne to the stand to testify to her observations about
Niles’ topcoat, but she refuses to testify. The prosecutor seeks to compel her
testimony.
No, she cannot be compelled.
no privilege communication
Witness spouse cannot be compelled to testify anything against
the defendant spouse.

(b) Assume Daphne is willing to testify against Niles. In addition to the topcoat
observation, she seeks to testify to the following: “Niles told me when he got home
that he stabbed Frazier.” Niles objects.
Spousal immunity privilege is inapplicable because she is willing
to testify.

But, here the confidential communication privilege can be invoked


by Niles. He can invoke the confidential communication privilege
with regard to the admission he made to her about stabbing his
brother which was a confidential communication made during the
privilege. Either spouse can invoke the privilege for confidential
communication.
54. EVIDENCE

HYPO 52. Assume that Daphne divorces Niles before his case goes to trial. The prosecutor
calls her to the stand.

(a) Can Daphne be compelled to testify to her observations about Niles’ topcoat?
Yes.
The observation of topcoat is not confidential.
As far as spousal immunity is concerned, that privilege is
over.Spousal immunity can only be use when the marriage exists.

(b) Can Niles prevent Daphne from disclosing his admission to her about stabbing
Frazier?
Yes. Because the admission to her was a confidential communication
made during the marriage.

VI. HEARSAY

A. Two-Part Definition
Out of court statement of a person
(1) _________________________________________________ (oral or written)
(The hearsay rule does not apply to machines (e.g., what a clock “said” about the
time) or to animals (e.g., the fact that a drug-sniffing dog barked at a suitcase).)
AND
the truth of the matter asserted in the statement
(2) Offered to prove _______________________________________

Hearsay rule: Hearsay is inadmissible unless an exception or exclusion applies.

Rationale: The credibility of the declarant (out-of-court speaker or author) at the


time the statement was made was not tested through cross-examination in the
presence of the current fact-finder

B. Non-Hearsay Statements

Some out-of-court statements may look like hearsay at first glance, but are not
hearsay if they are not offered to prove the truth of the matter asserted in the
statement. An out-of-court statement may be relevant to some issue simply
because it was spoken (or written). If offered for some other purpose, credibility
of the declarant is irrelevant. On the issue of whether the statement was spoken,
the witness on the stand can be cross-examined; or if the statement was in writing,
it can be examined as an exhibit.
EVIDENCE 55.

HYPO 53. Action by the estate of Percy against Damien seeking damages for the pain and
suffering Percy experienced in an auto accident caused by Damien. Damien denies liability and
also asserts that Percy died instantly in the accident. Witness on the stand proposes to testify that
shortly after the accident, Percy said, “Damien’s car ran the red light.”

(a) Hearsay if offered to prove who ran the red light?


Yes.

(b) Hearsay if offered to prove that Percy was alive following the accident?
Not hearsay

C. Principal Categories of Non-Hearsay Purposes

1. Verbal Act (Legally Operative Words). A situation where the substantive law
attaches rights and obligations to certain words simply because they were spoken.

HYPO 54. Gates sued Trump for breach of an oral contract. Witness takes the stand and
proposes to testify as follows: “I heard Trump say to Gates: ‘I accept your offer to sell
Microsoft.’” Hearsay?
No, verbal act. Words of contract formation.
Substantive law gives legal effects to certain
words, simple because these words were spoken.

Similar: terms of patent or copyright, making gift, bribe, perjury, fraud,


defamation, words accompanying ambiguous acts (e.g., D is charged with theft of
X’s car; D testifies, “As X handed me the keys, he said I could have the car for
the weekend.”).

2. To Show Effect on Person Who Heard or Read the Statement. If a person


hears someone else make certain statements, this may be relevant to put the
listener on notice of something, or to create fear, or to give the listener a motive or
probable cause to do something without regard to whether the statement is true.
56. EVIDENCE

HYPO 55. Plaintiff v. Supermarket. Plaintiff alleges she slipped and fell on a broken jar of
salsa in aisle 3 and that Supermarket had prior notice of the dangerous condition. Plaintiff’s
witness takes stand and proposes to testify: “Several minutes before Plaintiff entered aisle 3, I
heard another shopper tell Supermarket manager, ‘There’s a broken jar of salsa on the floor in
aisle 3.’” Inadmissible hearsay?

Depends on the purpose.


Inadmissible hearsay, if offered to prove that there
really was broken jar on the floor.
Admissible, if offered to prove the manager has
prior notice of potentially dangerous conditions.

HYPO 56. Sybil is charged with the murder of her husband Basil. To prove motive, the
prosecutor seeks to introduce an anonymous note to Sybil that was found in her possession at the
time of her arrest. The note stated, “Basil is having an affair with Polly.” Inadmissible hearsay?
No. The accusation about Basil shows potential motive.

3. Circumstantial Evidence of Speaker’s State of Mind

HYPO 57. Homer is prosecuted for murder. Defense: Insanity. Witness for Homer proposes to
testify: “Two days before the killing, Homer said, ‘I am Elvis Presley. It’s good to be back.’”
No. state of mind

Other examples: giving a false alibi implies consciousness of guilt; asking a


question may imply lack of knowledge.

D. Prior Statements of Trial Witness

HYPO 58. Prosecution of D for robbery. D takes the stand in his own defense and testifies: (a)
“I didn’t do it. (b) And I told the cops when they arrested me that I didn’t do it.” Should (a) and
(b) be excluded as hearsay?
a. not hearsay.
b. hearsay. To prove the truth of the matter.

General rule: A WITNESS’S OWN PRIOR STATEMENT, if offered to prove


the truth of the matter asserted in the statement, is hearsay and is
INADMISSIBLE unless an exception or exclusion applies.
jury wasn’t there.
EVIDENCE 57.

Three Witness-Statement Exclusions from Hearsay (called “exclusions” or “non-


hearsay”). Witness is currently subject to cross-examination AND:

identification of person (H37)


1. Witness’s prior statement of _____________________________; or

inconsistent
(H38)
2. Witness’s prior ___________________statement IF oral, under oath
and made during formal trial, hearing, proceeding or deposition; or

consistent (H48)
3. Witness’s prior _______________ statement to rebut charge of recent
fabrication, or contention of inconsistency or sensory deficiency.

E. Party Admissions (Statement of an opposing party)


an opposing party
1. Any statement made by ________________is admissible for its truth
against the opposing party
if it is offered___________________________.

2. Terminology: Called “exclusion” or “non-hearsay.” The Federal


Rules of Evidence were recently restyled. There were no substantive
changes to the Rules, but there were changes in terminology, the most
important of which is that “party admission” is now called “statement
of an opposing party.” The terms are substantively interchangeable
and either “admission” or “statement of an opposing party” may
appear as an answer choice on the MBE.

3. Theory: Party ought to bear the consequences of what she says. Can
explain to jury, and cannot complain about inability to cross-examine
self.

HYPO 59. X is charged with income tax evasion for the year 2000. Prosecutor wants to prove
X’s income during 2000, and offers into evidence a loan application X submitted to a bank in
that year. X objects on the ground that the loan application, which is filled with inflated
numbers, was self-serving and unreliable.
Party admission.
X’s own statement, offered against him.
Note: statement against interest hearsay exception later
58. EVIDENCE

HYPO 60. Ma v. Life Insurance Co. for non-payment of policy proceeds on the life of Pa.
Defense: Suicide. Defendant offers a letter by Ma to her friend in which she wrote, “When I
came home from shopping I found Pa dead on the floor with his revolver nearby. I didn’t see
what happened, but this was no accident. Pa did himself in.” Admissible despite Ma’s lack of
personal knowledge?
Party admission. Ma is a party, offered against her.

Adoptive Admission

If a party expressly or impliedly adopts a statement made by another person,


it is as though the party herself made the statement. Adoption by silence occurs
when a party who hears another person’s statement remains silent under
circumstances in which a reasonable person would protest if the statement were
false.

Vicarious Party Admission

Statement by agent/employee is admissible against principal/employer if


statement concerns matter within scope of agency/employment and is made
during the existence of the agency/employment relationship.

HYPO 61. Charlie the truck driver smashed into Pam’s house while on a run for Acme
Trucking, his employer. Charlie descended from the cab and calmly told Pam, “Sorry about
wrecking your home. I guess I took my eyes off the road. I was reaching down to get a beer and
a joint.” In Pam v. Acme, is Charlie’s statement admissible against Acme?
Yes, vicarious party admission.

HYPO 62. Thelma and Louise v. Acme Trucking for sex discrimination in failing to hire them.
They offer the statement of Charlie, an Acme truck driver, who told them over drinks one night,
“I know the Acme personnel office has a policy against hiring women no matter how qualified
they are.” Charlie’s statement is inadmissible because:

(A) Charlie was not on the job when he was speaking to Thelma and Louise.

(B) Charlie’s statement did not concern a matter within the scope of his employment.
EVIDENCE 59.

Co-conspirator’s Statement

The statement of a co-conspirator is admissible against a party who was a member


of the conspiracy if the statement was made (1) during and (2) in furtherance of
the conspiracy.

F. Hearsay Exceptions

1. Forfeiture by wrongdoing
2. Former testimony
3. Statement against interest
4. Dying declaration
5. Excited utterance
6. Present sense impression
7. Present state of mind
8. Declaration of intent
9. Present physical condition
10. Statement for purpose of medical treatment or diagnosis
11. Business records
12. Public records

(Also, recall Past Recollection Recorded, Learned Treatises)

Note on Criminal Defendant’s Right of Confrontation:

The Sixth Amendment right of confrontation requires that the criminal


defendant be “confronted” with the witnesses against him. Confrontation
cross-examination
is the opportunity for ____________________________ by the defendant.

Rule: In the context of hearsay, the prosecution may not use a hearsay
statement against the criminal defendant (even if it falls within a hearsay
exception) if:
testimonial
(1) the statement is _________________________________,

unavailable
(2) the declarant is _________________________________, and

(3) the defendant has had no opportunity for


cross-examination
________________________________________ (cross-
examination requirement may be satisfied either before or at
trial).
60. EVIDENCE

The meaning of “testimonial” statements is still being developed, but the


following applications have been established:
testimonial
(1) Grand jury testimony is ________________

(2) Statements in response to police interrogation:

(a) Testimonial if the primary purpose of the questioning is to


past events
establish or prove _____________________________

potentially relevant to
later criminal prosecution
___________________________________.

Example: Police arrive at domestic abuse crime scene,


arrest the abuser, and sit down with the victim to ask “what
happened?” At defendant’s trial, the victim refuses to
testify. The victim’s statements to the police, even if they
fall within a hearsay exception (e.g., excited utterances),
cannot be used against the defendant because they are
testimonial, the witness is now unavailable, and there has
been no cross-examination.

(b) Non-Testimonial if the primary purpose of the questioning


enable police assistance
is to _________________________________ to meet an
ongoing emergency
__________________________________.

Example: Domestic abuse victim calls 911 to seek help


while abuser is in other room with plans to resume the
abuse.

“Ongoing emergency” includes a situation in which the


crime has recently ended, the perpetrator is armed, and he
still poses a threat to the victim, the police, or the public at
large.
EVIDENCE 61.

(3) Documents:
non-testimonial
(a) Business records are ____________________.
(e.g., bank or phone company records).

testimonial
(b) Sworn Affidavits are _________________.
testimonial
(c) A forensic laboratory report is _________________
accuse a targeted individual
if its primary purpose is to ______________________________
of criminal conduct.
testimonial
Examples: (1) Analysis of drugs seized from a particular suspect
to ascertain if the drugs are cocaine, etc. (2) Analysis of the blood
of a suspected DWI driver to ascertain his blood alcohol content.testimonial
nontestimonial
statement
But a DNA report is __________________ if it analyzes a sample
of bodily fluid collected from a crime scene for the purpose of
no particular person suspected
developing a DNA profile if ___________________________
at the time of the analysis.

Even if a forensic report is testimonial, no confrontation


violation occurs if the prosecutor calls a testifying expert who
performed an independent analysis of the data, and the testifying
expert only generally refers to the report to show a partial basis for
her opinion without reading the report to the jury or introducing it
as an exhibit. Then the report is not being used for a hearsay
purpose. p.34

1. “Forfeiture Hearsay Exception” (Declarant Unavailable Due to


Defendant’s Wrongdoing): Any type of hearsay statement is admissible
against a defendant whose wrongdoing made the witness unavailable if the
court finds:
By a proponderance of evidence
(1) __________________________________________

(2) that defendant’s wrongdoing _____________________________


was designed to prevent a witness from testifying
_______________________________________________________
62. EVIDENCE

(By making the witness unavailable through his own wrongdoing,


defendant forfeits both the hearsay and Sixth Amendment
objection.)

Example: Tony Soprano is prosecuted for loan-sharking. Paulie


made incriminating statements about Tony to the police and during
a grand jury proceeding. Tony learned that Paulie was going to be
the key government witness at trial; and the next day Paulie’s dead
body was found in the river. The court determines by a
preponderance of the evidence that Tony arranged for Paulie’s
disappearance in order to prevent Paulie from testifying. Paulie’s
grand jury testimony and interview statements to the police are
admissible against Tony even though the statements are
testimonial, Paulie is unavailable, and Tony had no opportunity for
cross-examination.

2. Former Testimony

(a) The former testimony of a now-unavailable witness, if given at a


former proceeding or in a deposition, is admissible against a party
who, on the prior occasion, had an opportunity and motive to cross-
examine or develop the testimony of the witness. Issue in both
proceedings must be essentially the same.

(b) Theory: reliability assured by cross-examination on prior occasion;


however, we prefer live testimony, so witness must now be
unavailable.

HYPO 63. Bus accident. Passengers A and B were seriously injured. A sued Bus Co., alleging
negligence by bus driver. At trial, Witness testified for A that bus driver was intoxicated at time
of accident. Thereafter, Witness died. B now sues Bus Co. and seeks to admit a transcript of
Witness’s former testimony.
Admissible hearsay: former testimony
EVIDENCE 63.
secret: p, Witness, juror,no judge, no lawyer,
no cross
HYPO 64. Same bus accident. At grand jury, Witness testified that bus driver was intoxicated
at time of accident. Thereafter, Witness died. Bus driver is prosecuted for DWI. Prosecutor
seeks to admit a transcript of Witness’s grand jury testimony.
No.
Even though the witness is unavailable, the same issue, the bus driver,
against whom the former testimony is being offered, the bus driver had no
opportunity to cross exam the witness at prior proceeding.
It also violates the confrontation clause.

NOTE: If a grand jury witness testified to something favorable for the defendant,
and then became unavailable, the defendant might be able to use the former
testimony against the prosecution because the prosecutor, at the grand jury, did
have an opportunity, and usually the motive, to develop the witness’s testimony.

(c) Grounds of Unavailability:


Privilege
(1) ______________________________
Absence from the jurisdiction
Witness cannot be found with due
(2) ______________________________
diligence/witness is beyond court
subpoena power
______________________________
Illness or death
(3) ______________________________
Lack of memory
(4) ______________________________
A stubborn refusal to testify
(5) ______________________________

Note: Same grounds of unavailability apply to all exceptions where


unavailability is a requirement.

3. Statement Against Interest

(a) An unavailable declarant’s statement against his or her


1. pecuniary interest
________________
2. proprietary interest
3. penal interest
________________

________________

(money, property, or criminal liability)


64. EVIDENCE

(b) Theory: Not likely to lie when making a personally damaging


statement as to such interests.

(c) Statement against interest differs from party admission in four ways:

must be against interest when made

any person (not merely party) can make statement against interest

personal knowledge is required

declarant must be unavailable

HYPO 65. Plaintiff v. Acme Trucking, based on Charlie the truck driver’s negligent driving.
Charlie was fired immediately after the accident. Two weeks later, Charlie told Plaintiff’s
insurance adjuster that he had been drunk while driving. At trial, Charlie refused to testify on the
ground of self-incrimination. The insurance adjuster may properly testify to Charlie’s statement
as evidence against Acme because the statement is:

(A) A vicarious party admission. When the statement was made, Charlie was no longer
employee of Acme.

(B) A statement against interest.


against pecuniary interest, penal interest
unavailable because of the privilege

Note: Statement of interest is relevant can be used against any


party.

(d) Qualification in criminal cases: Statement against penal interest must


be supported by
circumstances showing trustworthiness of the statement
_______________________________________________________.

HYPO 66. Prosecution of Doppler for arson of Town Hall. Doppler calls Waldo to testify that
while sitting in a bar, Waldo heard Stranger say, “I’m the guy who torched Town Hall, but I’m
glad they think it’s Doppler. Just to be safe, I’m leaving town tomorrow.” Doppler’s attorney
demonstrates that Stranger has not been located despite diligent efforts to find him.

no corroborating evidence here.


EVIDENCE 65.

4. Dying Declaration

(a) Statement made under a belief of impending and certain death by a


now-unavailable declarant concerning the cause or surrounding
circumstances of the declarant’s death.

(b) Theory: Expectation of imminent death is a solemn occasion.

(c) Type of case:


Charge of homicide
Criminal cases: ________________
Any type of civil case
Civil cases: ___________________

HYPO 67. Prosecution of Dagger Dan for the murder of Victor Victim. A passerby found
Victor lying in the gutter in a pool of blood with a knife in his stomach. Victor told the passerby,
“It’s not looking too good for me. Dagger Dan did it, and I’m going to get him for this.” Victor
died an hour later. May the passerby testify to Victor’s statement as a dying declaration?
No. No impending imminent death.

Maybe excited utterance.

HYPO 68. Prosecution of Dillinger for bank robbery. At the scene, a bank officer spoke with
wounded Teller Tim, who gasped, “I’m a dead man. Get me a priest. Dillinger shot me as he
made his getaway.” Tim then lapsed into a coma from which he has not emerged. May the
officer testify to Tim’s statement as a dying declaration?
No. Bank robbery prosecution, not homicide.

Note: coma is ok to satisfy the unavailability requirement

HYPO 69. Same event except civil action against Dillinger for Tim’s personal injury damages.
Tim is still in a coma. Is Tim’s statement admissible as a dying declaration?
Yes.

Note on Confrontation: A dying declaration made to a police officer may be


testimonial, but it most likely qualifies as an exception to the confrontation requirement
of cross-examination. In dicta, the Supreme Court has noted that the admissibility of
dying declarations to law enforcement officials was well established when the Sixth
Amendment was adopted, meaning that dying declarations were intended to be exempt
from the confrontation rule.
66. EVIDENCE

“Spontaneous” Statements (Unavailability Not Required) for the remaining

5. Excited Utterance

(a) Statement concerning a startling event and made while declarant is still
under the stress of excitement caused by the event.

(b) Theory: excitement suspends one’s capacity to fabricate.

HYPO 70. Ernie observes a horrific head-on auto collision and excitedly tells a cop, who arrives
10 minutes later, “Oh my God, Officer! Both of those cars were going 90 miles an hour!” May
the cop properly testify to Ernie’s statement at a trial based on the accident?
Excited utterance.
Ernie observed a startling event .
Factors to consider in determining whether a statement qualifies as an excited utterance:
The nature of the event
(1) _____________________________
The passage of time
(2) _____________________________
Visual clues
(3) _____________________________

Exclamatory phrase
(a) ____________________
Excitement oriented verb
(b) ____________________
Exclamation point !
(c) ____________________

6. Present Sense Impression

(a) Description of an event made while the event is occurring or


immediately thereafter.
external event

(b) Theory: declarant has no time to fabricate

HYPO 71. Mom telephones her son Victor Victim at his apartment. “Mom,” he says, “Can you
wait a minute? Someone’s at the door.” Thirty seconds later, Victor gets back on the phone and
says, “Mom, can’t talk now. My new friend, Hannibal Lecter, is here for dinner. Call you later.”
The next day, the remains of Victor’s dead body are found in his apartment. Hannibal is on trial
for the murder. May Mom testify that Victor identified Hannibal as his dinner guest that night?
Yes.Victor made a present sense impression.
EVIDENCE 67.

7. Present State of Mind internal

(a) Contemporaneous statement concerning declarant’s present state of


mind, feelings, emotions.

(b) Theory: contemporaneous statement about matter as to which


declarant has unique knowledge.

HYPO 72. Probate of Wanda’s Will, in which she left all her money to the local pet cemetery.
Wanda’s family challenges the will on the ground that Wanda was insane when she executed it.
Pet cemetery offers testimony that a few days before execution of the will, Wanda said to her
friend, “I do not love my family anymore.” Admissible over hearsay objection?
Admissible. That is a reflection of her state of mind at the time she
was speaking.

8. Declaration of Intent

(a) Statement of declarant’s intent to do something in the future, including


the intent to engage in conduct with another person

(b) Theory: contemporaneous statement about matter as to which


declarant has unique knowledge

HYPO 73. Susan has died and her family sues Life Insurance Co. for nonpayment of the policy
proceeds. Defense: Suicide. Life Insurance Co. seeks to introduce a note found in Susan’s
apartment (in Susan’s handwriting) in which she said, “I’m going to end it all next week.”
Relevant.
Intent to do sth in the future.

HYPO 74. Prosecution of Raymond for murder of Vic. Before going out Monday night, Vic
told wife, “I’m meeting Raymond tonight at the bowling alley.” Vic’s dead body was found
Tuesday morning outside the bowling alley.
Admissible That was a declaration intent with the participation of other
people.
68. EVIDENCE

9. Present Physical Condition

(a) Statement made to anyone about declarant’s current physical condition

(b) Theory: contemporaneous statement about matter as to which


declarant has unique knowledge

HYPO 75. Plaintiff, whose arm was broken in accident with Defendant, sues for damages for
pain and suffering. Plaintiff may, of course, testify about the pain she experienced. But Plaintiff
also calls Neighbor to testify, (a) “I was with Plaintiff last July when she said, ‘I’m feeling a lot
of pain in my arm’ and again in December when she said (b) ‘I sure did feel a lot of pain in my
arm last July.’” Admissible over hearsay objections?
A. admissible. The neighbor was testifying to the declarant’s statements
of present physical condition.
B. Not admissible. Backward statement.

10. Statement Made for the Purpose of Obtaining Medical Treatment or


Diagnosis

(a) Statement made to anyone (usually to medical personnel) for the


purpose of obtaining medical treatment or diagnosis (including a
diagnosis for expert testimony) if it concerns the declarant’s
Present symptoms
1) ___________________ or
Past symptoms
2) ___________________ or
The general cause of the condition
3) ______________________________, but not including statements
describing the details of liability or the identity of a tortfeasor, unless it
is the identity of the abuser in a domestic abuse or child abuse case.

(b) Theory: A patient or injured person has a motive to be honest and


accurate to get medical assistance.

(c) This exception does not include oral statements made by a physician to
the patient. (Distinguish written entries made by a physician in
business records (e.g., hospital or office records).)
EVIDENCE 69.

HYPO 76. Plaintiff v. Defendant for pain-and-suffering damages based on alleged accident at
Defendant’s store. At trial, Plaintiff calls Dr. Treat, her treating physician, to testify, “When
Plaintiff came to see me for treatment a year after the accident, she said, (a) ‘The pain in my arm
is killing me. (b) I’ve been losing sleep at night for the past six months because of the pain in my
arm. (c) This all started when I fell down the stairway—(d) the one with no treads at
Defendant’s store.’” Admissible over hearsay objections?

a,b,c are all admissible as the statement made for the purpose of
obtaining medical treatment and diagnoses.

d is no good. Detail of liability and identity of tortfeasor.

Variation: Would statements (a), (b) and (c) be admissible if the statements had
been made to Dr. Whizkid, a physician she retained solely for the purpose of
testifying as an expert witness?

Medical diagnose includes the purpose of giving experts


testimony.

11. Business Records

(a) Elements:
business of any
type
(1) Records of a ______________
the regular course
(2) made in the ____________________ of business
routinely keeps
(3) the business __________________ such records
contemporaneously
(4) made ___________________
(at or about the time of the event recorded)

(5) contents consist of—


employees
information observed by ____________of the business, or

a statement that falls within an independent hearsay


exception
70. EVIDENCE

(b) Theory: Businesses depend on accurate, up-to-date record-keeping,


and accuracy is likely when employees are under a business duty to
make such records. Useful substitute for testimony of employees.

HYPO 77. Civil action by Pedestrian against Hot Rod Kid for recklessly running him down. At
trial, Pedestrian seeks to introduce the report of Officer McNulty, who arrived at the scene 10
minutes after the accident. The report, which was prepared by McNulty at the scene, states:

(a) “Upon arrival, I measured skid marks 50 feet in length.”


Admissible as business record
Police is a type of business
made at the time of event
info observed by officier

(b) “Officer Bunk, who witnessed the accident, told me that Hot Rod Kid was driving
nearly 60 miles per hour.”
Admissible as business record
Fact was reported to McNulty by a fellow cop

(c) “Bill Bystander told me that he saw the accident and that Hot Rod Kid ran through
the stop sign.”

Inadmissible. Not from fellow employee, from a outsider of the


business.
Outside has no business duty and incentive to record accurately.

(c) Proving Business Records Foundation:

(1) Call sponsoring witness to testify to the five elements of


business records hearsay exception; witness need not be author
of report—can be records custodian or any other
knowledgeable person within the business, or

(2) Written certification under oath attesting to elements of


business records hearsay exception (with advance notice to
opposing party).
EVIDENCE 71.

12. Public Records

(a) Records of a public office or agency setting forth:

(1) the activities of the office or agency (e.g., payroll records); or

(2) matters observed pursuant to a duty imposed by law (e.g., Weather


Bureau records of temperature); or

(3) findings of fact or opinion resulting from an investigation


authorized by law (e.g., OSHA inspection report on safety
conditions of factory).

(b) Exclusion:
prosecutorial purposes
Police reports prepared for ________________________ not civil
are not admissible against the defendant in a criminal case. Nor is the
prosecution in such cases allowed to introduce such reports against the
defendant under the alternative theory of business records.

Hearsay Within Hearsay

1. If a hearsay statement is included within another hearsay statement, the


evidence is inadmissible unless each statement falls within a hearsay
exception. (For purposes of this rule, a hearsay exemption (e.g., statement
of an opposing party) is treated like a hearsay exception.)

2. Examples: (1) If witness testifies, “A told me what B said,” the evidence


is inadmissible unless, for example, A’s statement is an excited utterance
made in response to B’s excited utterance. (2) In a homicide prosecution
against D, the victim, X, knowing he was about to die, said, “D did this to
me, and D said he was glad he did it as he ran away.” X’s statement is a
dying declaration (level one) that contains the statement of the opposing
party D (party admission) (level two).
72. EVIDENCE

Impeachment of Hearsay Declarants

Any impeachment method may be used to attack the credibility of a


hearsay declarant whose statement was admitted into evidence. If the
impeachment consists of a prior inconsistent statement, the usual
requirement that the declarant be given an opportunity to explain or deny
is waived.

HYPO 78. Shooter is on trial for the murder of Victim. In his hospital bed, Victim told the
nurse, “I’m feeling pretty good considering Billy Ray tried to kill me.” The next day, Victim
told a visitor, “I know I’m about to die. Shooter’s the one who shot me.” Victim then died.

(a) May the prosecution introduce Victim’s statement to the visitor over a hearsay objection?
Yes, statement about Shooter is a dying declaration.

(b) If the prosecutor is allowed to introduce Victim’s statement to the visitor, may Shooter
introduce Victim’s statement to the nurse in which he accused Billy Ray?
Cannot use the accusation against Billy Ray as substantive evidence
hearsay, not dying declaration.

Accusation against Billy Ray can be used as inconsistent statement of


declarant, impeaching victim’s credibility.

You might also like